An 80-mile trip is represented on a gridded map by a directed line segment from point M(3, 2) to point N(9,
13). What point represents 60 miles into the trip? Show your work and explain your reasoning.

Answers

Answer 1

Answer:

Step-by-step explanation:

If the distance from M to N is 80 miles and we want to find the coordinates of the point 60 miles into the trip, we are looking for the point 3/4 of the way from M to N, since 60 is 3/4 of 80. This is the process:

First we need to consider that MN is a directed vector. We first find the components of the directed vector, which is found in the change in x and the change in y. First step, then, looks like this:

<Δx, Δy> = <9-3, 13-2> = <6, 11> We will call those the x and y components of the vector (which comes from vector study in both physics and math, so if you don't understand that, it's ok! Just follow the process here and you'll be fine). Knowing that 60 is 3/4 of the way from M to N, we find 3/4 of both the x and y components, like this:

[tex]<\frac{3}{4}(6), \frac{3}{4}(11)>[/tex] which will give us 3/4 of the change in x and 3/4 of the change in y:

[tex]<\frac{18}{4},\frac{33}{4}>=<4.5,8.25>[/tex]

Since we are going 3/4 of the way from M to N, we add that component to the x and y coordinates of M, giving us 3/4 of the way from M to N, which translates to 60 miles of the 80 mile trip:

(3 + 4.5, 2 + 8.25) = (7.5, 10.25)

Those are the coordinates of the point that represents 60 miles into the 80 mile trip.


Related Questions

In In 5x + In In (x - 1) = 2

Answers

Answer:

exact form: x=-1/2

decimal form: x=-0.5

P(x) is a polynomial. here are a few values of p(x).
P(-5) = - 2
P(-3) = 6
P(3) = 7
P(5) = -1
What is the remainder when P(x) is divided by (x+5)?
What is the remainder when P(x) is divided by (x-3)?

Answers

Given:

Values of a polynomial P(x).

[tex]P(-5)=-2[/tex]

[tex]P(-3)=6[/tex]

[tex]P(3)=7[/tex]

[tex]P(5)=-1[/tex]

To find:

The remainder when P(x) is divided by (x+5).

The remainder when P(x) is divided by (x-3).

Solution:

If a polynomial P(x) is divided by (x-a), then the remainder is P(a).

If the polynomial P(x) is divided by (x+5), then the remainder is P(-5).

[tex]P(-5)=-2[/tex]

Therefore, the remainder is -2 when P(x) is divided by (x+5).

If the polynomial P(x) is divided by (x-3), then the remainder is P(3).

[tex]P(3)=7[/tex]

Therefore, the remainder is 7 when P(x) is divided by (x-3).

plz help me with this math and also explain

Answers

Step-by-step explanation:

[1]

SI = $250Rate (R) = 12[tex] \sf \dfrac{1}{2}[/tex] %Time (t) = 4 years

[tex]\longrightarrow \tt { SI = \dfrac{PRT}{100} } \\ [/tex]

[tex]\longrightarrow \tt { 250 = \dfrac{P \times 12\cfrac{1}{2} \times 4}{100} } \\ [/tex]

[tex]\longrightarrow \tt { 250 = \dfrac{P \times \cfrac{25}{2} \times 4}{100} } \\ [/tex]

[tex]\longrightarrow \tt { 250 = \dfrac{P \times 25 \times 2}{100} } \\ [/tex]

[tex]\longrightarrow \tt { 250 = \dfrac{P \times 50}{100} } \\ [/tex]

[tex]\longrightarrow \tt { 250 \times 100 = P \times 50} \\ [/tex]

[tex]\longrightarrow \tt { 25000 = P \times 50} \\ [/tex]

[tex]\longrightarrow \tt { \dfrac{25000}{50} = P } \\ [/tex]

[tex]\longrightarrow \underline{\boxed{ \green{ \tt { \$ \; 500 = P }}}} \\ [/tex]

Therefore principal is $500.

__________________

[2]

2/7 of the balls are red.3/5 of the balls are blue.Rest are yellow.Number of yellow balls = 36

Let the total number of balls be x.

→ Red balls + Blue balls + Yellow balls = Total number of balls

[tex]\longrightarrow \tt{ \dfrac{2}{7}x + \dfrac{3}{5}x + 36 = x} \\ [/tex]

[tex]\longrightarrow \tt{ \dfrac{10x + 21x + 1260}{35} = x} \\ [/tex]

[tex]\longrightarrow \tt{ \dfrac{31x + 1260}{35} = x} \\ [/tex]

[tex]\longrightarrow \tt{ 31x + 1260= 35x} \\ [/tex]

[tex]\longrightarrow \tt{ 1260= 35x-31x} \\ [/tex]

[tex]\longrightarrow \tt{ 1260= 4x} \\ [/tex]

[tex]\longrightarrow \tt{ \dfrac{1260 }{4}= x} \\ [/tex]

[tex]\longrightarrow \underline{\boxed{ \tt { 315 = x }}} \\ [/tex]

Total number of balls is 315.

A/Q,

3/5 of the balls are blue.

[tex]\longrightarrow \tt{ Balls_{(Blue)} =\dfrac{3 }{5}x} \\ [/tex]

[tex]\longrightarrow \tt{ Balls_{(Blue)} =\dfrac{3 }{5}(315)} \\ [/tex]

[tex]\longrightarrow \tt{ Balls_{(Blue)} = 3(63)} \\ [/tex]

[tex]\longrightarrow \underline{\boxed{ \green {\tt { Balls_{(Blue)} = 189 }}}} \\ [/tex]

solve the simultaneous equation: x-y=2
xy=36​

Answers

Answer:

Y= –7.08, 5.08

Step-by-step explanation:

hope ya ready bro.

X=36/Y

replace 36/y instead of X

36/Y–Y=2===> 36–Y²=2Y===> Y²+2Y–36=0

Y1= –1+√37≈ –7.08

Y2= –1–√37≈ 5.08

In which section of the number line is 32−−√?

Answers

where's the number line?

maybe u can attach it at the comments:)

Answer:

Section B

Step-by-step explanation:

What is 6.273 rounded to the nearest thousandths?

Answers

Answer:

6.270

Step-by-step explanation:

3 is below 5 so you just turn it into a 0. If it was 5 or above you would just add a number to the 7.

Find the measure of the missing angle using the exterior angle sum theorm.

Answers

Answer:

85°

Step-by-step explanation:

The exterior angle of a triangle is=sum of the opposite interior angles

So

? °=45°+40°

If f(x) = 2x2 + 1, what is f(x) when x = 3?

Answers

Answer:

18

Step-by-step explanation:

f(x)=2x(2+1)

f(3)=2(3)(2+1)

    =6(2+1)

    =12+6

    =18

Need help ASAP !!!when creating a question for a survey , the way a question is written:

Answers

Answer:

The third one i think

Step-by-step explanation:

Since a survey shouldn't be biased or anything.

a^2×c^2/c^2×d^2+bc/ad reduce the algebraic ​

Answers

Answer:

[tex]\frac{a^2*c^2}{c^2*d^2}+\frac{bc}{ad}= \frac{a^3 + bcd}{ad^2}[/tex]

Step-by-step explanation:

Given

[tex]\frac{a^2*c^2}{c^2*d^2}+\frac{bc}{ad}[/tex]

Required

Simplify

We have:

[tex]\frac{a^2*c^2}{c^2*d^2}+\frac{bc}{ad}[/tex]

Cancel out [tex]c^2[/tex]

[tex]\frac{a^2*c^2}{c^2*d^2}+\frac{bc}{ad}= \frac{a^2}{d^2}+\frac{bc}{ad}[/tex]

Take LCM

[tex]\frac{a^2*c^2}{c^2*d^2}+\frac{bc}{ad}= \frac{a^3 + bcd}{ad^2}[/tex]

A lending library has a fixed charge for the first three days and an additional charge for each day thereafter. Saritha paid ₹27 for a book kept for seven days, while Susy paid ₹21 for the book she kept for five days. Find the fixed charge and the charge for each extra day.​​

Answers

Answer:

Ji Gud afternoon.

I'm fine

what about you madam.

Hope u r fine and stay safe and healthy

The equation d = m/v can be used to calculate the density,d, of an object with mass,m, and volume,V. Which is an equivalent equation solver for V?

Answers

Answer:

v = m/d

Step-by-step explanation:

since

[tex]d = \frac{m}{v} \: \\ \\ v = \frac{m}{d} [/tex]

If
5
3 cosα = in the first quadrant, what does cot α
equal?

Answers

Answer:

5/4

Step-by-step explanation:

To Find :-

cot a .

Solution :-

By question ,

=> cos a = 5/3 = b/h

=> p = √ 5² - 3² = √ 25 -9 = 4

Therefore ,

=> cot a = b/p = 5/4

The sum of three consecutive odd numbers is 63. What are the numbers?​

Answers

19,21,23. Adding all of these consecutive odd numbers is equal to 63

Answer:

19, 21 and 23

Step-by-step explanation:

→ Make an algebraic expression for the 3 consecutive numbers

2x + 1, 2x + 3 and 2x + 5

→ Add the expressions together and make it equal to 63

2x + 1 + 2x + 3 + 2x + 5 = 63

→ Simplify

6x + 9 = 63

→ Minus 9 from both sides

6x = 54

→ Divide both sides by 6

x = 9

→ Resubstitute x = 9 into the 3 expressions

x = 9 into 2x + 1 is 19

x = 9 into 2x + 3 is 21

x = 9 into 2x + 5 is 23

I would raise it to 1grand but I’m very broke

Answers

Answer:

ITS A LOL

Step-by-step explanation:

help help help pls :)

Answers

Answer:

[tex]opposite\approx 70.02[/tex]

Step-by-step explanation:

The triangle in the given problem is a right triangle, as the tower forms a right angle with the ground. This means that one can use the right angle trigonometric ratios to solve this problem. The right angle trigonometric ratios are as follows;

[tex]sin(\theta)=\frac{opposite}{hypotenuse}\\\\cos(\theta)=\frac{adjacent}{hypotenuse}\\\\tan(\theta)=\frac{opposite}{adjacent}[/tex]

Please note that the names ([tex]opposite[/tex]) and ([tex]adjacent[/tex]) are subjective and change depending on the angle one uses in the ratio. However the name ([tex]hypotenuse[/tex]) refers to the side opposite the right angle, and thus it doesn't change depending on the reference angle.

In this problem, one is given an angle with the measure of (35) degrees, and the length of the side adjacent to this angle. One is asked to find the length of the side opposite the (35) degree angle. To achieve this, one can use the tangent ([tex]tan[/tex]) ratio.

[tex]tan(\theta)=\frac{opposite}{adjacent}[/tex]

Substitute,

[tex]tan(35)=\frac{opposite}{100}[/tex]

Inverse operations,

[tex]tan(35)=\frac{opposite}{100}[/tex]

[tex]100(tan(35))=opposite[/tex]

Simplify,

[tex]100(tan(35))=opposite[/tex]

[tex]70.02\approx opposite[/tex]


Find (a) the mean, (b) the median, and
(c) the mode of the number of verses in
the first eight chapters of Proverbs.
Proverbs 1 23 4 5 6 7 8
Verses 33 22352723352736
(a)
(b)
(c)

Answers

Answer:

Mean = 33+ 22+35+27+23+35+27+36 = 238/8 = 29.75

Median = 27+23 = 50/2 = 25

Mode = 35

Range = 36 - 22 = 14

Step-by-step explanation:

Stay Blessed

Arvin has $10000 to invest. He invests part in a term deposit paying 5%/year, and the remainder in Canada savings bonds paying 3.4%/year. At the end of the year, he earned simpler interest of $413. How much did he invest at 5%/year?

Answers

Answer:

$4,562.5

Step-by-step explanation:

The amount Arvin has to invest, P = $10,000

The interest paid on the investment in the term deposit = 5%/year

The interest paid om the investment in Canada savings bonds = 3.4%/year

The amount Arvin earned at the of the year as simple interest, A = $413

Let, x, represent the amount Arvin invested in the term deposit and let, y, represent the amount he invested in Canada savings bonds, we can get the following system of equations

x + y = 10,000...(1)

0.05·x + 0.034·y = 413...(2)

Making y the subject of equation (1) and substituting the value in equation (2), we get;

From equation (1), we get, y = 10,000 - x

Plugging the above value of y in equation (2) gives;

0.05·x + 0.034 × (10,000 - x) = 413

∴ 0.05·x - 0.034·x + 340 = 413

x = (413 - 340)/(0.05 - 0.034) = 4,562.5

Therefore, the amount Arvin invested in the term deposit at 5%, x = $4,562.5

(y = 10,000 - x

∴ y = 10,000 - 4,562.5 = 5,437.5

The amount Arvin invested in Canada savings bonds, y = $5,437.5.)

find the volume of the rectangular prism. plz answer this lol

Answers

Answer:

.....how when the dimensions are not even clear lol

Answer:

48 cm³

Step-by-step explanation:

the volume of a rectangular prism= length × breadth × height

= 8× 3 × 2

= 48 cm³

Find the value of y

Help please

Answers

Answer:

6

Step-by-step explanation:

Set your formula up as

15 = 2y+3

15 - 3 = 2y

12 = 2y

12 / 2 = y

6 = y

The measure of an angle is eight times the measure of its supplementary angle. What is the measure of each angle?

Answers

Answer:

160°

Step-by-step explanation:

Let one angle be x

Let second angle be 8x

Sum of supplementary angles = 180°

x+ 8x = 180°

9x = 180

x = 180/9 = 20

First angle= x= 20°

Second angle = 8x = 8*20 = 160°

The polygons are similar, but not necessarily drawn to scale. Find the value of x. PLEASE HELPPPP

Answers

Answer:

x = 27.5.

Step-by-step explanation:

There are given numbers on each side. If the figures are similar, then they have a set ratio for each value.

So, 55:8 and x:4. If you want to, you can flip it, so that it is 8:55 and 4:x.

With that in mind, it is easy to see what the ratio is. Because 4 is half of 8, x is half of 55. 55 divided by 2 is 27.5.

Therefore, x = 27.5.

if u help me i will give you brainliest <3

Answers

Answer:

a) 6

b) 12:35 ( not so sure about this one)

Step-by-step explanation:

a)

distance = speed * time

ruths distance = 4* 1.5 (because it takes her 1h 30 mins)

distance = 6miles    

b)  1/4 here is quarter so, 15mins

10:30 and then add 50 mins to that.

     she leaves at 11:20 and takes 1h and 15 mins ( 75mins) to walk home

     so 11:20 + 1h 15min

     = 12:35

hope this helps :)

Answer:

a) 6 miles

b) 12:35 pm

Step-by-step explanation:

a) 9 am to 10:30 am = 1 hour and 30 mins = 1.5 hours

4mph x 1.5 = 6miles

b) 10:30 am + 50 mins = 11:20 am

1 hour = 60 mins

60/4 = 15 mins

1 1/4 = 1 hour 15 mins

11:20 am + 1 hour 15 mins = 12:35 pm

Someone please help me ASAP

Answers

Step-by-step explanation:

a vector multiplied by a scalar is equal to it's image. The expression above gives an equation and after solving, it gives you the image

Does anyone know the answer to this question?

Answers

The answer is A; g(x) = |x+4| - 2

For the function G defined by G(x)=5x+3, find G(r+5)

Answers

Given function:

g(x) = 5x + 3

Find

g(r+5)

Substitute x with r = 5:

g(r + 5) = 5(r + 5) + 3 = 5r + 25 + 3 = 5r + 28

Answer:

G ( r + 5 ) = 5r + 28

Step-by-step explanation:

Given ;

G ( x ) = 5x + 3

To Find :-

G ( r + 5 )

Solution :-

plug r + 5 as x in the function.

G ( r + 5 ) = 5 ( r + 5 ) + 3

distribute 5

G ( r + 5 ) = 5r + 25 + 3

combine like terms

G ( r + 5 ) = 5r + 28

what is the value of b in this equation?

A. -20
B. -6
C. 6
D. 20

Answers

Answer:

Step-by-step explanation:

a- 12

b- 2

c- 11

and the next part is

x- 6

y- 18

Answer:

B

Step-by-step explanation:

Using the rules of exponents

[tex](a^m)^{n}[/tex] = [tex]a^{mn}[/tex]

[tex]a^{-m}[/tex] = [tex]\frac{1}{m}[/tex]

Then

[tex](y^b)^{4}[/tex] = [tex]\frac{1}{y^{24} }[/tex]

[tex]y^{4b}[/tex] = [tex]y^{-24}[/tex]

Equate the exponents

4b = - 24 ( divide both sides by 4 )

b = - 6 → C

log8-log4 ÷ log4-log2=





Answers

The answer is log(4)-1

Which point is in the solution set of this system inequalities?

A. (0,0)

B. None of these

C. (5,1)

D. (3,7)

Answers

Answer:

B

Step-by-step explanation:

To find which ordered pairs are solutions to the inequalities we can simply plug in the x and y values of the ordered pairs into the inequalities and if the equation is true for both inequalities then the ordered pair is a solution to the inequalities.

For (0,0)

x = 0

y = 0

y > x + 5

Substitute 0 for y and x

0 > 0 + 5

Simplify right side

0 > 5

The inequality is not true as 5 is greater than 0, not less than. So immediately we can eliminate answer choice A.

For (5,1).

x = 5

y = 1

y > x + 5

Substitute 5 for x and 1 for y

1 > 5 + 5

Simplify right side

1 > 10

Again, the equation is not true as 1 is not greater than 10. This means that c cannot be the answer

For (3,7)

x = 3

y = 7

y > x + 5

Substitute 3 for x and y for 7

7 > 3 + 5

Simplify right side

7 > 8

7 is not greater than 8 meaning that (3,7) cannot be a solution to the inequalities

None of the ordered pairs created true equations hence the answer is B

a 10 foot ladder rests against a vertical wall if the bottom of the ladder slides away from the wall at a speed of 2 ft/s how fast is the angle betwween the top of the ladder and the wall changing when that angle is

Answers

Answer:

d∅/dt = √2/5 Rad/sec

Step-by-step explanation:

According to the Question,

Given That, a 10-foot ladder rests against a vertical wall if the bottom of the ladder slides away from the wall at a speed of 2 ft/s how fast is the angle between the top of the ladder and the wall changing when that angle is π/4.

Solution,

Let x be the Distance between the base of the wall and the bottom of the ladder.

and let ∅ be the angle between the top of the ladder and the wall.

Then, Sin∅ =x/10  so, x=sin∅ *10

Differentiating with respect to time t we get,

dx/dt = 10 * cos∅ * d∅ /dt

We have given that dx/dt = 2 ft/s and ∅ =π/4

Now, Put these value we get

2 = 10 *(cos(π/4))* d∅/dt

2 = 10/√2 * d∅/dt

d∅/dt = √2/5 Rad/sec

Other Questions
9. Rewrite the following sentences changing the voice:1. Did you buy the new English MCB? MOTS2. Let the door be opened. [LOTS]3. Cows eat grass. [LOTS]4. Farmers depend on Monsoon to sow the seeds. MOTS5. Teachers will have prepared a detailed note of the lesson. HOTS6. We must obey the rules. HOTS7. Crops were harvested before the rain by them. [LOTS] how to solve for resistors In the historic U.S. Supreme Court case of Marbury v. Madison(1803), Chief Justice John Marshall A. established judicial review for the SCOTUS B. saved the institutional integrity of the Court by avoiding the appearance of impotence C. ruled that statutory law cannot change constitutional law D. ruled an act of Congress unconstitutional for the first time E. all of the above how does this show the relationship between photosynthesis and cellular respiration g In the global stage of a firm's globalization, ________. A. the need for training is high B. training is focused on local culture and interpersonal skills C. the need for training is virtually nonexistent D. host-country nationals are trained to understand parent-country products and policies Please help ASAP!!! Thank you!!! NEED ANSWER QUICK!!Camillo needs 2,400 oz of jelly for the food challenge. If 48 oz of jelly cost $3.84, how much will Camillo spend on jelly? Explain how you can find your answer. VINE BUSCANDO ORO Y ENCONTRE PIEDRAS Simple percent question!!!!My question says that line segment DC is 50% longer than line segment BD, and BD is 40 ft. Does that mean DC is 60 ft or 80? Please explain, brainliest to whoever explains it properly and quickly nimas letter to her brother class 9 what are produced by ovaries Which of the following best represents the accomplishments of Spain's golden century? Help please guys if you dont mind 9. Which property is not important when selecting a material to use as a light bulb filament?dhigh melting pointhigh vapor pressurehigh ductility pls help me with this The 100th term of 8, 8^4, 8^7, 8^10, The diagram shows a cylinder of diameter 6 cm and height 20 cm what is the volume in cm3 The expression 2(a+b)-8.1 for certain values of a and b.find the value of the following expressions for the same values of a and b: 3(a+b) If you could do any job from home, what would do? Why? How do you explain Meiosis?